Diễn Đàn MathScopeDiễn Đàn MathScope
  Diễn Đàn MathScope
Ghi Danh Hỏi/Ðáp Thành Viên Social Groups Lịch Ðánh Dấu Ðã Ðọc

Go Back   Diễn Đàn MathScope > Sơ Cấp > Đại Số và Lượng Giác

News & Announcements

Ngoài một số quy định đã được nêu trong phần Quy định của Ghi Danh , mọi người tranh thủ bỏ ra 5 phút để đọc thêm một số Quy định sau để khỏi bị treo nick ở MathScope nhé !

* Nội quy MathScope.Org

* Một số quy định chung !

* Quy định về việc viết bài trong diễn đàn MathScope

* Nếu bạn muốn gia nhập đội ngũ BQT thì vui lòng tham gia tại đây

* Những câu hỏi thường gặp

* Về việc viết bài trong Box Đại học và Sau đại học


Trả lời Gởi Ðề Tài Mới
 
Ðiều Chỉnh Xếp Bài
Old 02-04-2011, 02:18 PM   #151
haimap27
+Thành Viên+
 
haimap27's Avatar
 
Tham gia ngày: Jul 2010
Bài gởi: 99
Thanks: 136
Thanked 44 Times in 34 Posts
Trích:
Nguyên văn bởi phantiendat_hv View Post
Bài 61:
$
\sqrt{\frac{a}{b+\sqrt{ab}}}+\sqrt{\frac{b}{c+ \sqrt{bc}}}+\sqrt{\frac{c}{a+\sqrt{ca}}}\ge\frac{3 }{2} $
vì $ab,bc,ca \ge 0 $ nêna,b,c cùng dấu$\Rightarrow \frac{a}{b},\frac{b}{c},\frac{c}{a}\ge 0 $
$VT=\sum\sqrt{\frac{\sqrt{\frac{a}{b}}}{1+\sqrt{\fr ac{ b}{a}}}} $
đặt $m=\sqrt{\frac{a}{b}},n=\sqrt{\frac{b}{c}},p=\sqrt{ \frac{c}{a}}(m,n,p \ge 0,mnp=1) $
$VT=\sum\sqrt{\frac{m}{\frac{1}{m}+1} = \sum\frac{m}{\sqrt{m+1}} $
đặt $m=x^3,n=y^3,p=z^3 $
$VT=\sum\frac{x^3}{\sqrt{x^3+1}}=\sum\frac{x^3}{\sq rt{(x+1)(x^2-x+1)}} \ge \sum\frac{2x^3}{x^2+2} $
bây giờ ta chỉ cần C/m: $\sum\frac{x^3}{x^2+2} \ge \frac{3}{4} $
Áp dụng Cauchy-Schwarz, ta có:
$\sum\frac{x^3}{x^2+2} \ge \frac{(x^2+y^2+z^2)^2}{x^3+y^3+z^3+2(x+y+z)} $
$\frac{(x^2+y^2+z^2)^2}{x^3+y^3+z^3+2(x+y+z)} \ge \frac{3}{4} $
$\Leftrightarrow 4(x^4+y^4+z^4)+8xyz(x+y+z) \ge 3(x^3+y^3+z^3)+6(x+y+z) $
$\Leftrightarrow 4(x^4+y^4+z^4)+2(x+y+z) \ge 3(x^3+y^3+z^3) $
Áp dụng AM-GM,ta có:
$\sum(x^4+x^4+x) \ge \sum(3x^3) $
$\Rightarrow $đpcm.
[RIGHT][I][B]Nguồn: MathScope.ORG[/B][/I][/RIGHT]
 
__________________
I WILL DO IT=p

thay đổi nội dung bởi: novae, 02-04-2011 lúc 05:34 PM Lý do: latex
haimap27 is offline   Trả Lời Với Trích Dẫn
The Following 2 Users Say Thank You to haimap27 For This Useful Post:
phantiendat_hv (03-04-2011), Quydo (02-04-2011)
Old 02-04-2011, 07:40 PM   #152
duynhan
+Thành Viên+
 
Tham gia ngày: Dec 2009
Bài gởi: 231
Thanks: 103
Thanked 118 Times in 68 Posts
Trích:
Nguyên văn bởi batigoal View Post
Bài 60(Batigoal)
Cho 3 ba số thực $x,y,x>0 $ thỏa mãn $xyz=1 $.Chứng minh rằng:

$\frac{x^3y}{x^3y+2}+\frac{y^3z}{y^3z+2}+\frac{z^3x }{z^3x+2}\geq 1 $
$a= x^3y, b = y^3z, c = z^3 x $
$\Rightarrow abc=1 $

$\Large \left{ \begin{matrix}
a = \frac{x}{y} & & \\
b =\frac{y}{z} & & \\
c = \frac{z}{x} & & \\
\end{matrix} \right.
$
Bất đẳng thức cần chứng minh tương đương với :
$\sum_{cyc} \frac{x}{x+2y} \ge 1 $
$VT = \sum_{cyc} \frac{x^2 }{x^2 + 2xy} \ge 1 $
[RIGHT][I][B]Nguồn: MathScope.ORG[/B][/I][/RIGHT]
 
__________________
duynhan is offline   Trả Lời Với Trích Dẫn
Old 02-04-2011, 07:48 PM   #153
batigoal
Super Moderator
 
batigoal's Avatar
 
Tham gia ngày: Jul 2010
Đến từ: Hà Nội
Bài gởi: 2,895
Thanks: 382
Thanked 2,968 Times in 1,295 Posts
Bai62 Cho 3 số thực dương $a,b,c $.Chứng minh rằng:

$\frac{a^2+bc}{a^2+(b+c)^2}+\frac{b^2+ca}{b^2+(c+a) ^2}+\frac{c^2+ab}{c^2+(a+b)^2}\leq \frac{18}{5}\frac{a^2+b^2+c^2}{(a
+b+c)^2} $

[RIGHT][I][B]Nguồn: MathScope.ORG[/B][/I][/RIGHT]
 
__________________
“ Sức mạnh của tri thức là sự chia sẻ tri thức”

[Only registered and activated users can see links. ]

thay đổi nội dung bởi: batigoal, 07-04-2011 lúc 12:10 AM Lý do: nội dung
batigoal is offline   Trả Lời Với Trích Dẫn
Old 02-04-2011, 08:14 PM   #154
Kratos
+Thành Viên+
 
Tham gia ngày: Nov 2009
Đến từ: Toán 0912, PTNK, Tp.HCM
Bài gởi: 87
Thanks: 25
Thanked 160 Times in 73 Posts
Gửi tin nhắn qua Yahoo chát tới Kratos
Bài 63. Cho các số thực $a, b, c $ thoả mãn $a^2+b^2+c^2=3 $. Tìm GTLN của biểu thức
$P=a^3+b^3+c^3+a^2b+b^2c+c^2a $
[RIGHT][I][B]Nguồn: MathScope.ORG[/B][/I][/RIGHT]
 
Kratos is offline   Trả Lời Với Trích Dẫn
The Following User Says Thank You to Kratos For This Useful Post:
n.v.thanh (12-08-2011)
Old 03-04-2011, 10:05 AM   #155
Eragon1994
+Thành Viên+
 
Eragon1994's Avatar
 
Tham gia ngày: Feb 2011
Đến từ: KSA-HCMC
Bài gởi: 106
Thanks: 28
Thanked 74 Times in 51 Posts
Gửi tin nhắn qua Yahoo chát tới Eragon1994
Tặng các bác trên mathscope 1 bài mình chế(hiển nhiên là chỉ sử dụng BĐT Cauchy-Schwarzt)
Bài 64
Cho $a,b,c>0 $ thỏa $a+b+c=1 $.Chứng minh rằng:
$\sum \frac{ab}{c^2(a+b)^2} \ge \sum \frac{1}{ab+c} $
[RIGHT][I][B]Nguồn: MathScope.ORG[/B][/I][/RIGHT]
 
__________________
Học không chơi đánh rơi tuổi trẻ
Chơi không học vừa khỏe vừa vui


thay đổi nội dung bởi: Eragon1994, 03-04-2011 lúc 11:30 AM Lý do: đánh theo thứ tự
Eragon1994 is offline   Trả Lời Với Trích Dẫn
The Following User Says Thank You to Eragon1994 For This Useful Post:
hizact (03-04-2011)
Old 03-04-2011, 01:29 PM   #156
leviethai
+Thành Viên+
 
Tham gia ngày: Nov 2008
Đến từ: Thành phố Hồ Chí Minh. Nhưng quê tôi là Ninh Bình.
Bài gởi: 513
Thanks: 121
Thanked 787 Times in 349 Posts
Gửi tin nhắn qua Yahoo chát tới leviethai
Trích:
Nguyên văn bởi vthiep94 View Post
mình xin nhận xét thế này. Nếu có quá lời các bạn bỏ quá cho:
Hầu hết những bài bất đẳng thức nêu ra đều trong sách anh Cẩn và có lời giải. Mình nghĩ các bạn nên kiếm nhiều hơn những bài lạ 1 chút ví dụ như những bài đã được giải rồi theo phương pháp khác và giải lại theo cách CS, thì mới nói lên ứng dụng lớn lao của bdt này.
Mình sẽ giải 1 bài đã post rồi (giải bằng cách khác ) và giải theo cách CS
[Only registered and activated users can see links. ]
Đây là cách giải của mình
Bất đẳng thức tương đương với
$\[
(4 - (ab)^2 )(4 - (bc)^2 )(4 - (ac)^2 ) \ge (2 + ab)(2 + bc)(2 + ac)
\] $
Ta chứng minh bổ đề $\[
a + b + c \ge (ab)^2 + (bc)^2 + (ac)^2
\]
$ (1)
Thật vậy $\[
\left( {a + b + c} \right)^2 (a^4+ b^4 + c^4) \ge (a^2 + b^2 + c^2 )^3 = 27;
\]
$;
Ta cần chứng minh $\[
\left( {ab+bc+ac} \right)^2 (a^4 + b^4 + c^4) \ge (a^2 + b^2 + c^2 )^3 = 27
\]
$
thật vậy $\[
(a^4 + b^4 + c^4 )\left[ {(ab)^2 + (bc)^2 + (ac)^2 } \right]^2 \le \frac{{(a^2 + b^2 + c^2 )^6 }}{{27}} = 27
\] $;
Theo (1) ta dễ dàng chứng minh được $\[
(4 - (ab)^2 )(4 - (bc)^2 )(4 - (ac)^2 ) \ge (4 - a)(4 - b)(4 - c)
\] $;
vậy ta cần chứng minh
$\[
(4 - a)(4 - b)(4 - c) \ge (2 + ab)(2 + bc)(2 + ac)
\] $
Ta cần chứng minh
$\[
\begin{array}{l}
4 - a)(4 - b)(4 - c) \ge (2 + ab)(2 + bc)(2 + ac) \\
\Leftrightarrow (8 - 2a)(8 - 2b)(8 - 2c) \ge (4 + 2ab)(4 + 2bc)(4 + 2ac); \\
\Leftrightarrow \prod {\left[ {4 + (a - 1)^2 + b^2 + c^2 } \right]} \ge (4 + 2ab)(4 + 2bc)(4 + 2ac) \\
\end{array}
\]
$
mà $\[
\begin{array}{l}
\prod {\left[ {4 + (a - 1)^2 + b^2 + c^2 } \right]} \ge \prod {(4 + } b^2 + c^2 ); \\
\prod {(4 + } b^2 + c^2 ) \ge \prod {(4 + 2ab)} \\
\end{array}
\]
$ Ta chứng minh xong bài toán !
Mình hoàn toàn đồng ý với bạn và vô cùng vui khi còn nhiều người suy nghĩ như vậy.

Tuy nhiên, không biết bạn chứng minh bất đẳng thức này thế nào
Trích:
$\[
(4 - (ab)^2 )(4 - (bc)^2 )(4 - (ac)^2 ) \ge (4 - a)(4 - b)(4 - c)
\] $
vì với $a=0,b=c=\sqrt{\dfrac{3}{2}} $, bất đẳng thức không đúng nữa.

Bài này hiện mình chưa tìm được lời giải bằng C-S, nhưng có lời giải đưa về 1 biến bằng Dirichlet cũng khá thú vị.

@Eragon1994:
Bài 64. Dễ thấy
$ab+c=(c+a)(c+b) $
Nên bất đẳng thức tương đương
$\sum\limits_{sym} {\frac{{ab}}{{{{(ca + cb)}^2}}}} \ge \frac{{2(a + b + c)}}{{(a + b)(b + c)(c + a)}}, $
mà ta lại có, theo Cauchy Schwarz và Nesbit (chứng minh bằng C-S)
$\left[ {\sum\limits_{sym} {ab} } \right]\left[ {\sum\limits_{sym} {\frac{{ab}}{{{{(ca + cb)}^2}}}} } \right] \ge {\left[ {\sum\limits_{sym} {\frac{{ab}}{{ca + cb}}} } \right]^2} \ge \frac{9}{4}, $
nên ta chỉ cần chứng minh
$\frac{9}{{4(ab + bc + ca)}} \ge \frac{{2(a + b + c)}}{{(a + b)(b + c)(c + a)}} $
bất đẳng thức này tương đương với
$a(b-c)^2+b(c-a)^2+c(a-b)^2\ge 0. $
Vậy ta có điều phải chứng minh. $\hfill \Boxed $

Mình xin nhắc lại 2 bài sau

Bài 59. Cho $a,b,c,d $ là các số thực dương. Chứng minh rằng
$2(ac+bd)(a^2+b^2+c^2+d^2)\ge (a+b+c+d)(abc+bcd+cda+dab). $

Bài 63. Cho $a,b,c $ là các số thực thỏa mãn $a^2+b^2+c^2=3 $. Tìm giá trị lớn nhất của biểu thức
$P=a^3+b^3+c^3+a^2b+b^2c+c^2a. $

Đây là hai bài có cách phát biểu đơn giản, nhưng lại hay.
[RIGHT][I][B]Nguồn: MathScope.ORG[/B][/I][/RIGHT]
 
leviethai is offline   Trả Lời Với Trích Dẫn
The Following 4 Users Say Thank You to leviethai For This Useful Post:
hizact (03-04-2011), huynhcongbang (04-04-2011), Quydo (03-04-2011), vthiep94 (03-04-2011)
Old 03-04-2011, 09:26 PM   #157
vthiep94
+Thành Viên+
 
Tham gia ngày: Dec 2009
Bài gởi: 197
Thanks: 185
Thanked 49 Times in 31 Posts
Bạn Hải cho mình lời giải bằng diriclet được ko ?
[RIGHT][I][B]Nguồn: MathScope.ORG[/B][/I][/RIGHT]
 
vthiep94 is offline   Trả Lời Với Trích Dẫn
Old 03-04-2011, 09:55 PM   #158
Persian
+Thành Viên+
 
Tham gia ngày: Nov 2010
Đến từ: Có những thứ mình đã nhẫn tâm đánh mất sẽ không bao giờ lấy lại được.
Bài gởi: 257
Thanks: 103
Thanked 200 Times in 112 Posts
Trích:
Nguyên văn bởi leviethai View Post
nên ta chỉ cần chứng minh
$\frac{9}{{4(ab + bc + ca)}} \ge \frac{{2(a + b + c)}}{{(a + b)(b + c)(c + a)}} $
bất đẳng thức này tương đương với
$a(b-c)^2+b(c-a)^2+c(a-b)^2\ge 0. $
Vậy ta có điều phải chứng minh. $\hfill \Boxed $
Cho mình hỏi cái BĐT trên cậu có cách pt tích nào nhanh ko hay chỉ là phá hết ra rồi nhóm vào -chắc phải còn cách khác nếu ta ko thích biến đổi phức tạp
[RIGHT][I][B]Nguồn: MathScope.ORG[/B][/I][/RIGHT]
 
Persian is offline   Trả Lời Với Trích Dẫn
Old 03-04-2011, 10:09 PM   #159
leviethai
+Thành Viên+
 
Tham gia ngày: Nov 2008
Đến từ: Thành phố Hồ Chí Minh. Nhưng quê tôi là Ninh Bình.
Bài gởi: 513
Thanks: 121
Thanked 787 Times in 349 Posts
Gửi tin nhắn qua Yahoo chát tới leviethai
@Persian: thực ra không có gì gọi là phức tạp ở đây cả, nếu bung hết ra một cách có đối xứng (không nên khai triển hết ra mà hãy cố nhóm các đại lượng đối xứng hoặc hoán vị, ta sẽ khai triển nhanh hơn và dễ nhìn hơn), bạn sẽ thấy phân tích bình phương là vô cùng tự nhiên. Nếu không, có cách dễ nhìn hơn
$(a+b+c)(ab+bc+ca)=abc+(a+b)(b+c)(c+a)\le \dfrac{9}{8}(a+b)(b+c)(c+a) $

@vthiep94: có 2 cách nhưng đều giống nhau ở chỗ giả sử $(1-ab)(1-ac)\ge 0 $ hay $1+a^2bc\ge ab+ac $. Ta có

$(2-ab)(2-ac)=2(1-ab)(1-ac)+2-a^2bc\ge 2-a^2bc \ge 2-\dfrac{a^2(b^2+c^2)}{2}=2-\dfrac{a^2(3-a^2)}{2} $
hơn nữa
$2-bc\ge 2-\dfrac{b^2+c^2}{2}=2-\dfrac{3-a^2}{2}=\dfrac{1+a^2}{2}, $
Vậy ta chỉ cần chứng minh
$[2-\dfrac{a^2(3-a^2)}{2}][1+a^2]\ge 2 $
tương đương với
$a^2(a^2-1)^2\ge 0 $
Ta có điều phải chứng minh. $\hfill \Box $

[RIGHT][I][B]Nguồn: MathScope.ORG[/B][/I][/RIGHT]
 

thay đổi nội dung bởi: leviethai, 03-04-2011 lúc 10:12 PM
leviethai is offline   Trả Lời Với Trích Dẫn
The Following 2 Users Say Thank You to leviethai For This Useful Post:
Persian (03-04-2011), vthiep94 (03-04-2011)
Old 04-04-2011, 03:30 PM   #160
phantiendat_hv
+Thành Viên Danh Dự+
 
phantiendat_hv's Avatar
 
Tham gia ngày: Sep 2010
Đến từ: Bình Phước.....$ xứ bụi $
Bài gởi: 379
Thanks: 276
Thanked 410 Times in 185 Posts
Gửi tin nhắn qua Yahoo chát tới phantiendat_hv
Bài 65:

$
\frac{\sqrt{ab+4bc+4ca}}{a+b}+
\frac{\sqrt{bc+4ca+4ab}}{b+c}+
\frac{\sqrt{ca+4ab+4bc}}{c+a}\ge\frac{9}{2} $
[RIGHT][I][B]Nguồn: MathScope.ORG[/B][/I][/RIGHT]
 
__________________
Phan Tiến Đạt
phantiendat_hv is offline   Trả Lời Với Trích Dẫn
Old 04-04-2011, 04:22 PM   #161
Eragon1994
+Thành Viên+
 
Eragon1994's Avatar
 
Tham gia ngày: Feb 2011
Đến từ: KSA-HCMC
Bài gởi: 106
Thanks: 28
Thanked 74 Times in 51 Posts
Gửi tin nhắn qua Yahoo chát tới Eragon1994
Liệu có tồn tại lời giải bằng BDT Cauchy-Schwarzt cho bài sau không:
Problem 66
Cho $x,y,z>0 $.Chứng minh rằng:
$\frac{\sqrt{x+y}}{z}+\frac{\sqrt{y+z}}{x}+ \frac{\sqrt{z+x}}{y} \ge \frac{4(x+y+z)}{\sqrt{(x+y)(y+z)(z+x)}} $
--------------------------------------------------------------------
P/s:Bài này mình phải xài kết hợp 2 BDT là Cauchy-SchwraztSchur bậc 5 nên giờ post lên đây để xin lời giải hoàn toàn bằng BDT Cauchy-Schwarzt
[RIGHT][I][B]Nguồn: MathScope.ORG[/B][/I][/RIGHT]
 
__________________
Học không chơi đánh rơi tuổi trẻ
Chơi không học vừa khỏe vừa vui


thay đổi nội dung bởi: phantiendat_hv, 04-04-2011 lúc 04:29 PM Lý do: Latex
Eragon1994 is offline   Trả Lời Với Trích Dẫn
Old 05-04-2011, 09:55 AM   #162
phantiendat_hv
+Thành Viên Danh Dự+
 
phantiendat_hv's Avatar
 
Tham gia ngày: Sep 2010
Đến từ: Bình Phước.....$ xứ bụi $
Bài gởi: 379
Thanks: 276
Thanked 410 Times in 185 Posts
Gửi tin nhắn qua Yahoo chát tới phantiendat_hv
Trích:
Nguyên văn bởi Eragon1994 View Post
Liệu có tồn tại lời giải bằng BDT Cauchy-Schwarzt cho bài sau không:
Problem 66
Cho $x,y,z>0 $.Chứng minh rằng:
$\frac{\sqrt{x+y}}{z}+\frac{\sqrt{y+z}}{x}+ \frac{\sqrt{z+x}}{y} \ge \frac{4(x+y+z)}{\sqrt{(x+y)(y+z)(z+x)}} $
--------------------------------------------------------------------
P/s:Bài này mình phải xài kết hợp 2 BDT là Cauchy-SchwraztSchur bậc 5 nên giờ post lên đây để xin lời giải hoàn toàn bằng BDT Cauchy-Schwarzt
Bài này mình dùng chebychev là ổn còn Cauchy-Schwarzt thì..........
[RIGHT][I][B]Nguồn: MathScope.ORG[/B][/I][/RIGHT]
 
__________________
Phan Tiến Đạt
phantiendat_hv is offline   Trả Lời Với Trích Dẫn
Old 05-04-2011, 11:41 AM   #163
kandten
+Thành Viên+
 
Tham gia ngày: Mar 2011
Bài gởi: 15
Thanks: 6
Thanked 7 Times in 5 Posts
[QUOTE=Eragon1994;89035]
Bài này em dùng cauchy_swchars và cauchy mọi người giùm em xem có đúng không
$\Leftrightarrow \sum \frac{(x+y)\sqrt{(x+z)(z+y)}}{z}\geq\sum \frac{(x+y)(\sqrt{xy}+z)}{z}=\sum\frac{(x+y)\sqrt{ xy}}{z}+(x+y)\geq \sum \frac{2xy}{z}+(x+y)\geq 4(x+y+z) $
Lại có: $\sum \frac{2xy}{z}=\sum \frac{xy}{z}+\frac{yz}{x}\geq 2(x+y+z) $
Từ đó --> đpcm
[RIGHT][I][B]Nguồn: MathScope.ORG[/B][/I][/RIGHT]
 
kandten is offline   Trả Lời Với Trích Dẫn
The Following User Says Thank You to kandten For This Useful Post:
haimap27 (05-04-2011)
Old 05-04-2011, 01:22 PM   #164
Kelacloi
+Thành Viên+
 
Tham gia ngày: Mar 2011
Bài gởi: 252
Thanks: 50
Thanked 164 Times in 114 Posts
Bài 59
$(a+b+c+d)(abc+bcd+cda+dab)=[(a+c)+(b+d)][ ac(b+d)+bd(a+c)]=(ac+bd)[(a+c)(b+d)+ac(b+d)^2+bd(a+c)^2 $
Áp dụng am-gm , ta có :
*$(a+c)(b+d) \le \frac{1}{2}((a+c)^2+(b+d)^2}\le a^2+b^2+c^2+d^2 $
*$ac(b+d)^2+bd(a+c)^2=ac(b-d)^2+bd(a-c)^2+8abcd \le (ac+bd)[ (b-d)^2+(a-c)^2]+8abcd = (ac+bd)(a^2+b^2+c^2+d^2-2ac-2bd)+8abcd $
$=(ac+bd)(a^2+b^2+c^2+d^2)+8abcd-2(ac+bd)^2 \le (ac+bd)(a^2+b^2+c^2+d^2) $

Cộgn 2 bdt trên lại ta được điều phải chứng minh

Bài 63
Đặt :$p=a+b+c $.Ta có :
$ 3=a^2+b^2+c^2 \le (a+b+c)^2 \le 9 $
$\Rightarrow 3 \ge p \ge \sqrt{3} $
Ta có :
$P=\sum a^3+\sum a^2b=(\sum a^3+\sum a^2b+\sum a^2c)-\sum a^2c =(\sum a^2)(\sum a)-\sum a^2c $
$=3(\sum a)+\sum a-\sum c-\sum a^2c $
Theo am-gm , ta có :$\sum (c+a^2c) \ge \sum 2ac =(\sum a)^2-\sum a^2=p^2-3 $
HEnce, $P \le 4p-(p^2-3)=6+(p-1)(3-p) \le 6 $
[RIGHT][I][B]Nguồn: MathScope.ORG[/B][/I][/RIGHT]
 
__________________
Kelacloi is offline   Trả Lời Với Trích Dẫn
Old 05-04-2011, 01:33 PM   #165
Kratos
+Thành Viên+
 
Tham gia ngày: Nov 2009
Đến từ: Toán 0912, PTNK, Tp.HCM
Bài gởi: 87
Thanks: 25
Thanked 160 Times in 73 Posts
Gửi tin nhắn qua Yahoo chát tới Kratos
Trích:
Nguyên văn bởi Kelacloi View Post
Bài 63
Đặt :$p=a+b+c $.Ta có :
$ 3=a^2+b^2+c^2 \le (a+b+c)^2 \le 9 $
$\Rightarrow 3 \ge p \ge \sqrt{3} $
Ta có :
$P=\sum a^3+\sum a^2b=(\sum a^3+\sum a^2b+\sum a^2c)-\sum a^2c =(\sum a^2)(\sum a)-\sum a^2c $
$=3(\sum a)+\sum a-\sum c-\sum a^2c $
Theo am-gm , ta có :$\sum (c+a^2c) \ge \sum 2ac =(\sum a)^2-\sum a^2=p^2-3 $
HEnce, $P \le 4p-(p^2-3)=6+(p-1)(3-p) \le 6 $
a, b, c là 3 số thực nên bạn không thể dùng AM-GM như vậy được.

Và GTLN cũng không đạt tại 6.
[RIGHT][I][B]Nguồn: MathScope.ORG[/B][/I][/RIGHT]
 
Kratos is offline   Trả Lời Với Trích Dẫn
The Following 2 Users Say Thank You to Kratos For This Useful Post:
11112222 (22-05-2011), n.v.thanh (12-08-2011)
Trả lời Gởi Ðề Tài Mới

Bookmarks

Ðiều Chỉnh
Xếp Bài

Quuyền Hạn Của Bạn
You may not post new threads
You may not post replies
You may not post attachments
You may not edit your posts

BB code is Mở
Smilies đang Mở
[IMG] đang Mở
HTML đang Tắt

Chuyển đến


Múi giờ GMT. Hiện tại là 11:52 PM.


Powered by: vBulletin Copyright ©2000-2024, Jelsoft Enterprises Ltd.
Inactive Reminders By mathscope.org
[page compression: 110.22 k/126.35 k (12.76%)]